Past Exam Errors 1 Flashcards
A three-year-old girl has a large secundum atrial septal defect.
The magnitude of the left-to-right atrial shunt will be most influenced by the:
A. left atrial pressure.
B. left ventricular compliance.
C. pulmonary vascular resistance.
D. right ventricular compliance.
E. systemic vascular resistance
D - RV compliance
Uptodate:
The pathophysiology of isolated ASDs depends upon the relationship of pulmonary and systemic resistances, the compliance of the right and left ventricles, and the size of the defect.
With a small ASD, left atrial pressure is slightly higher than right atrial pressure, resulting in continuous flow of oxygenated blood from the left to the right atrium across the defect. The pressure gradient between the two atria and the amount of shunt flow depend upon the size of the defect and the relative distensibility of the right and left sides of the heart.
Even when the right and left atrial pressures are equal, as will be seen with a large defect, left-to-right shunting still occurs because of the greater compliance of the right ventricle compared with the left ventricle.
Can lead to right sided volume overload, although heart failure is rarely seen before 30 years of age.
Which one of the following diseases is not considered to be mediated by antigens acting as "super-antigens"? A. Kawasaki disease. B. Rheumatic fever. C. Scarlet fever. D. Staphylococcal toxic shock syndrome. E. Streptococcal toxic shock syndrome.
B - Rheumatic fever - note this question is from 1999 ?outdated, multiple sources now listing rheumatic fever as strep superantigen involvement…
Wikipedia
Superantigens (SAgs) are a class of antigens that result in excessive activation of the immune system. Specifically it causes non-specific activation of T-cells resulting in polyclonal T cell activation and massive cytokine release. SAgs are produced by some pathogenic viruses and bacteria.
Compared to a normal antigen-induced T-cell response where 0.0001-0.001% of the body’s T-cells are activated, these SAgs are capable of activating up to 20% of the body’s T-cells.
Diseases associated with superantigen production: Diabetes mellitus Eczema Guttate psoriasis Kawasaki disease Nasal polyps Rheumatic fever Rheumatoid arthritis Scarlet fever Toxic shock syndrome Infective endocarditis
Collaboration involving physical contact between B and T cells is essential for the activation of B cells and
immunoglobulin class switching. Contact occurs between CD40 on the surface of the B cell and the CD40 ligand
on activated T cells.
In which one of the following is CD40 ligand binding most likely to be defective?
A. Selective IgA deficiency.
B. Severe combined immunodeficiency.
C. Wiskott-Aldrich syndrome.
D. X-linked agammaglobulinaemia.
E. X-linked hyper IgM syndrome
E - Hyper IgM Syndrome
Uptodate
The hyperimmunoglobulin M (hyper-IgM or HIGM) syndromes include a heterogeneous group of conditions characterized by defective class-switch recombination (CSR), resulting in normal or increased levels of serum IgM associated with deficiency of immunoglobulin G (IgG), immunoglobulin A (IgA), and immunoglobulin E (IgE) and poor antibody function.
Most commonly due to X-linked disease of CD40L gene.
SCID: Defect in T and B cell development, severe, causes vary but cytokine receptor defects (IL2-R) and adenosine deaminase deficiency most common (intracellular accumulation of lymphocyte toxic metabolites).
CVID: Hypogammaglobulinaemia, defect in B-cell differentiation, cause unclear ?intrinsic B cell defect impairing T cell mediated activation.
Hyper IgE: AD/AR, recurrent skin/resp infections, eczema.
Selective IgA Def: MOST COMMON primary immunodeficiency, unclear significance, mostly asymptomatic, unknown aetiology
Wiskott-Aldrich: Triad of microthrombocytopenia, eczema, recurrent pyogenic infections. X-recessive. Mutation in WASP gene (Wiskott-Aldrich syndrome protein).
X-linked agammaglobulinaemia: severe deficiency of all classes, mutation in BTK gene on X chromosome -> ineffective Bruton’s tyrosine kinase -> failed maturation of B cells.
Which associated feature of attention deficit hyperactivity disorder at presentation is most likely to predict a poorer response to stimulant medication treatment? A. Age greater than 10 years. B. Co-existing anxiety disorder. C. Female sex. D. Prominent aggression. E. Severe inattentiveness.
B - Co-existing anxiety disorder
Which one of the following partial pressures of arterial oxygen would you expect from a normal one-year-old breathing 60% oxygen at sea level? A. 290 mmHg. B. 390 mmHg. C. 450 mmHg. D. 520 mmHg. E. 570 mmHg.
B - 390mmHg
The alveolar gas equation is used to calculate alveolar oxygen partial pressure:
PAO2 = (Patm - PH2O) FiO2 - PaCO2 / RQ
While PAO2 is the partial pressure of oxygen in the alveoli, Patm is the atmospheric pressure at sea level equaling 760 mm Hg. PH2O is the partial pressure of water equal to approximately 45 mm Hg. FiO2 is the fraction of inspired oxygen. PaCO2 is the carbon dioxide partial pressure in alveoli, which in normal physiological conditions is approximately 40 to 45 mm Hg, and the RQ (respiratory quotient - standard value 0.82).
At sea level without supplemented inspired oxygenation, the alveolar oxygen partial pressure (PAO2) is:
PAO2 = (760 - 47) 0.21 - 40 / 0.8 = 99.7 mm Hg
Estimating A-a gradient:
Normal A-a gradient = (Age + 10) / 4
A-a gradient increases 5 to 7 for every 10% increase in FiO2.
The principle mechanism which ensures excretion of excess sodium when the dietary intake of salt is high involves: A. atrial natriuretic factor. B. glomerular filtration. C. Na-K ATPase. D. renin-angiotensin pathway. E. tubulo-glomerular feedback.
A - ANP
Sodium is the primary cation in the ECF and determines ECF osmolarity.
Responses to REDUCED Na/BP/ECF
- baroreceptors detected reduced BP -> sympathetic nervous system activation and vasoconstriction
- RAAS -> reduced BP, reduced renal perfusion, juxtaglomerular apparatus secretes renin
Responses to INCREASED Na/BP/ECF
- natriuretic factors/peptides -> cause natriuresis. ANP = volume receptors in atria, BNP = volume receptors in brain ventricles
The laboratory telephones you with the following test results of blood taken from a 16-year-old male student who had requested hepatitis B screening. HBsAg (surface antigen) negative Anti-HBs (surface antibody) positive Anti-HBc (core antibody) positive The most likely explanation for these results is: A. acute hepatitis B infection. B. chronic hepatitis B infection. C. hepatitis D superinfection. D. past hepatitis B vaccination. E. previous hepatitis B infection.
E - previous infection
Surface antigen = current infection
Surface antibody alone = vaccination
Core antibody = previous exposure
E antigen = active replication
The anticoagulant effect of the coumarin derivative warfarin is most likely to be enhanced by which one of the following? A. Carbamazepine. B. Erythromycin. C. Oral contraceptive. D. Phenytoin. E. Vitamin K.
B - erythromycin
Key Messages:
- CYP3A4 is responsible for the metabolism of more than 50% of medicines.
- CYP3A4 activity is absent in new-borns but reaches adult levels at around one year of age.
- The liver and small intestine have the highest CYP3A4 activity.
- Some important CYP3A4 interactions are due to intestinal rather than hepatic enzyme inhibition (eg, grapefruit).
- There is considerable variability in CYP3A4 activity in the population.
- Women have higher CYP3A4 activity than men.
- Potent inhibitors of CYP3A4 include clarithromycin, erythromycin, diltiazem, itraconazole, ketoconazole, ritonavir, verapamil, goldenseal and grapefruit.
- Inducers of CYP3A4 include phenobarbital, phenytoin, rifampicin, St. John’s Wort and glucocorticoids.
Warfarin is a racemic mixture of stereo isomers, which are 99 percent bound to albumin. The drug is metabolized in the liver and kidneys, with the subsequent production of inactive metabolites that are excreted in the urine and stool.
Which one of the following is most important in recovery from acute bronchiolitis due to respiratory syncytial virus? A. IgG antibody production. B. IgM antibody production. C. Interferon. D. Natural killer cells. E. T lymphocytes.
E - T lymphocytes
A Weschler Intelligence Scale for Children (WISC) assesses general intelligence and its contributing factors.
Which one of the following requires an additional test in order to be adequately measured? A. Freedom from distractibility. B. Perceptual organisation. C. Reading ability. D. Speed of information processing. E. Verbal comprehension.
C - reading ability
Boast notes
Weschler
6-16 years
Involves • Verbal comprehension • Visual spatial • Fluid reasoning • Working memory • Processing speed
Good points
• Generates an IQ score
• Can be used to dx ADHD and learning disability
• 45-65 minutes to administer
Limitations
• Does NOT include reading ability
The cavernous sinus contains all of the following nerves except: A. abducens. B. facial. C. oculomotor. D. trigeminal. E. trochlear.
B - facial nerve
The cavernous sinus is one of the dural venous sinuses of the head. It is a network of veins that sit in a cavity, approximately 1 x 2 cm in size in an adult. The carotid siphon of the internal carotid artery, and cranial nerves III, IV, V (branches V1 and V2) and VI all pass through this blood filled space.
A patient with pancreatic insufficiency refuses to take enzyme replacement and later presents with renal calculi.
Increased excretion of which one of the following is the most likely cause of calculus formation?
A. Bicarbonate.
B. Cysteine.
C. Oxalate.
D. Phosphate.
E. Urate.
C - oxalate
Fat malabsorption causes increased binding of diet calcium by free fatty acids, reducing the calcium available to precipitate diet oxalate. Delivery of unabsorbed bile salts and fatty acids to the colon increases colonic permeability, the site of oxalate hyper-absorption in enteric hyperoxaluria.
FFA also bind magnesium -> hypomagnesaemia
In a four-year-old child, marked dilatation of the pulmonary artery would be most likely to occur in which one of the
following?
A. Atrial septal defect.
B. Bilateral pulmonary artery branch stenosis.
C. Idiopathic arterial calcification.
D. Mild pulmonary valve stenosis.
E. Severe pulmonary valve stenosis.
D - mild PS
Post stenotic dilatation
For which one of the following antiepileptic drugs is the measurement of plasma levels of most value in clinical management? A. Carbamazepine. B. Lamotrigine. C. Phenytoin. D. Sodium valproate. E. Vigabatrin.
D - valproate
?old question, lecture was pretty against routine monitoring of AEDs…
Antiepileptic drug-level monitoring has been a common clinical practice since the advent of antiepileptic drugs (AEDs). The practice of maintaining AEDs within laboratory-defined therapeutic ranges is a myth, with professionals overestimating values in most clinical settings. Epilepsy, in general, is a clinical diagnosis, with diagnostic modalities such as EEG and MRI considered to be complementary investigations. The assessment of AED efficacy should also be clinical and the AED level utilized as a complementary tool in selected situations.
Carbamazepine, valproate, and oral phenytoin should be tested after steady state has been reached, just before the next dose is given, and at a consistent time of day.
Cytochrome P450 (CYP) enzymes are responsible for the oxidation processes of phase I metabolism. CYP2C93 polymorphisms have been demonstrated to decrease the rate of metabolism of phenobarbital, phenytoin, and valproic acid.
UDP glucuronosyltransferase (UGT) enzymes
catalyze the glucuronidation of xenobiotics as part
of phase II drug metabolism. Polymorphisms of
UGT2B7 significantly decrease serum valproic acid
levels in epilepsy patients.
The relative potency of the glucocorticoid activity of betamethasone compared to prednisolone is closest to which one of the following ratios? A. 3:1. B. 6:1. C. 10:1. D. 15:1. E. 25:1.
B - 6:1
Comparied with cortisol (=1):
- cortisone 0.8
- prednisolone 4
- methylpred 5
- betamethasone and dexamethasone 25
Which one of the following primary immunodeficiency disorders is most likely to be associated with anaphylaxis to blood products? A. Ataxia-Telangiectasia. B. Common variable immunodeficiency. C. IgG subclass deficiency. D. Selective IgA deficiency. E. X-linked agammaglobulinaemia
D - IgA deficiency
Anaphylactic reactions are a rare complication during intravenous immune globulin (IVIG) administration. Patients with undetectable serum IgA are at increased risk, given the possibility of anti-IgA antibody formation and consequent sensitization to IgA contained in the IVIG preparation.
A 13-year-old boy has been diagnosed with insulin dependent diabetes for eight years and coeliac disease for four years. When he is reviewed, he complains of mild lethargy. He is noted to be more pigmented than normal.
Which one of the following tests would be most useful in diagnosing adrenal insufficiency in this situation?
A. Adrenal autoantibodies.
B. Plasma ACTH (adrenocorticotrophic hormone).
C. Plasma alpha MSH (melanocyte stimulating hormone).
D. Plasma cortisol.
E. Urinary free cortisol.
B - ACTH
Cushing syndrome/disease
ACTH to help distinguish central/primary/secondary causes. Cushing’s syndrome may be either corticotropin (ACTH) dependent or independent. Approximately 80 percent of endogenous Cushing’s syndrome cases are ACTH dependent, and approximately 20 percent are ACTH independent.
ACTH-dependent Cushing’s syndrome 80
Cushing’s disease 68 - most common
Ectopic ACTH syndrome 12
Ectopic CRH syndrome «1
ACTH-independent Cushing's syndrome 20 Adrenal adenoma 10 Adrenal carcinoma 8 Micronodular hyperplasia <1 Macronodular hyperplasia <1
Pseudo-Cushing’s syndrome
Major depressive disorder 1
Alcoholism
Which one of the following psychotropic drugs has the greatest alpha-adrenergic agonist effect? A. Clomipramine. B. Clonidine. C. Fluoxetine. D. Haloperidol. E. Thioridazine
B - clonidine
Alpha 1 mediates vasoconstriction, inc BP, etc.
Alpha 2 receptors in the brain stem and in the periphery inhibit sympathetic activity and thus lower blood pressure. Alpha 2 receptor agonists such as clonidine or guanabenz reduce central and peripheral sympathetic overflow and via peripheral presynaptic receptors may reduce peripheral neurotransmitter release.
Mechanism of Action
As an alpha-adrenergic agonist in the nucleus tractus solitarii (NTS), clonidine excites a pathway that inhibits excitatory cardiovascular neurons. Clonidine has an alpha-antagonist effect in the posterior hypothalamus and medulla.
The variance for height (or weight) in a population may be expressed in percentiles or standard deviation scores.
One standard deviation below the population mean (-1 SD) approximates which one of the following percentiles?
A. 3rd.
B. 10th.
C. 15th.
D. 25th.
E. 33rd.
C - 15th
First standard deviation either direction covers ~2/3 (68%) of population, leaving 1/3 ~32% left, but split at the top and bottom. Therefore aligns with the 16th percentile.
Which of the following immunoglobulins fix complement when they bind to antigen? A. Only IgG. B. IgG and IgA. C. IgG, IgA and IgM. D. IgG and IgM. E. Only IgM.
D - IgG, IgM
?Google search saying some evidence for IgA as well… 1999 question
“Recent advances have shown that also IgA is capable of activating the complement system.” - pubmed
USMLE book: Classic pathway—IgG or IgM mediated. Alternative pathway—microbe surface molecules. Lectin pathway—mannose or other sugars on microbe surface.
Which one of the following is predominantly excreted unchanged in the urine? A. Carbamazepine. B. Gabapentin. C. Lamotrigine. D. Phenytoin. E. Sodium valproate.
B - gabapentin
Enzyme-inducing antiepileptic drugs include: Carbamazepine. Eslicarbazepine acetate. Oxcarbazepine. Perampanel (at a dose of 12 mg daily or more). Phenobarbital. Phenytoin. …
Non-enzyme inducing antiepileptic drugs include: Acetazolamide. Clobazam. Clonazepam. Ethosuximide. Gabapentin.
Valproic acid differs from other older generation AEDs in being an inhibitor rather than an inducer of drug metabolizing enzymes.
Carbamazepine, phenytoin, phenobarbital and primidone (henceforth referred to collectively as enzyme-inducing AEDs) stimulate the activity of a variety of cytochrome P450 (CYP) enzymes.
The plasma half-life of nitric oxide is extremely short.
This is primarily due to:
A. binding of nitric oxide to haemoglobin.
B. binding of nitric oxide to plasma albumin.
C. metabolic degradation of nitric oxide within vascular muscle fibres.
D. uptake of nitric oxide by alveolar macrophages.
E. uptake of nitric oxide by alveolar type II cells.
A - binding to haemoglobin
Which one of the following drugs crosses the placenta to a negligible degree? A. Captopril. B. Digoxin. C. Pancuronium. D. Sodium valproate. E. Warfarin
C - pancuronium
Examination of the synovial fluid from a subject with rheumatoid arthritis reveals many polymorphs and the
following factors. Which one of these is likely to be predominantly responsible for attracting polymorphs into the synovial fluid?
A. Complement C4.
B. Complement C5a.
C. Interleukin 1.
D. Interleukin 2 receptors.
E. Tumour necrosis factor
B - complement C5a
The main function of complement is protecting the host from infection/inflammation by recruiting (chemotaxis) and enhancing phagocytosis by innate immunity, finally leading to lysis of the target cells.
USMLE book: C3b—opsonization. C3a, C4a, C5a—anaphylaxis. C5a—neutrophil chemotaxis. C5b-9—cytolysis by MAC. C3b binds bacteria.
“Hot T-bone stEAK”: IL-1: fever (hot). IL-2: stimulates T cells. IL-3: stimulates bone marrow. IL-4: stimulates IgE production. IL-5: stimulates IgA production. IL-6: stimulates aKute-phase protein production.
“Clean up on aisle 8.” Neutrophils are recruited
by IL-8 to clear infections.
If the full immunisation schedule has been properly administered to a two-year-old, which one of the following gives the least effective protection? A. Diphtheria. B Haemophilus influenzae type b. C. Measles. D. Pertussis. E. Rubella.
D - Pertussis
Pertussis vaccines are effective, but not perfect. They typically offer good levels of protection within the first 2 years after getting the vaccine, but then protection decreases over time. Public health experts call this ‘waning immunity. ‘ Similarly, natural infection may also only protect you for a few years.
?Waning immunity quicker for pertussis than others.
Receive additional DTP vax at 4 years ?due to waning.
Which one of the following is an endothelium-derived vasodilator? A. Angiotensin II. B. Endothelin 1. C. Prostacyclin. D. Prostaglandin H2. E. Thromboxane A2
C - prostacyclin
The endothelium-dependent regulation of vascular tone is predominantly by four major players: nitric oxide (NO), prostaglandin I2 (PGI2 = prostacyclin), endothelium-derived hyperpolarizing factor (EDHF), and endothelin-1 (ET-1).
In healthy vessels, the endothelium expresses constitutive forms of nitric oxide synthase (NOSIII) and cyclo-oxygenase (COX-1), which produce the vasoactive hormones NO and prostacyclin, respectively. Both NO and prostacyclin relax blood vessels and inhibit platelet activation.
Angiotensin 2 = constriction
Endothelin = contriction
PGH2 = the common precursor for a number of biologically important prostanoids
Thromboxane = produced in platelets, stimulates platelets and platelet aggregation
The mean pulmonary artery (capillary) wedge pressure most closely approximates which one of the following pressures? A. Left atrial mean. B. Pulmonary artery diastolic. C. Pulmonary vein wedge mean. D. Right atrial mean. E. Right ventricular end-diastolic.
A - left atrial mean
A three-month-old baby is found to be deaf. What is the most likely cause? A. Aminoglycoside toxicity. B. Congenital CMV infection. C. Congenital rubella infection. D. Inherited. E. Perinatal asphyxia.
D - inherited
SNHL (all answers are of this variety anyway)
Conductive hearing loss is caused by a mechanical problem of the outer or middle ear that interferes with conduction of sound to the inner ear. It can occur at any location from the outer ear (pinna, external auditory canal) to the stapes footplate and oval window. Examples include cerumen impaction, middle ear fluid, and ossicular chain fixation.
Sensorineural hearing loss (SNHL) results from damage, disease, or other disorders affecting the inner ear (cochlea) and/or the auditory nerve. SNHL can be categorized as congenital (including both hereditary and nonhereditary causes) and acquired.
Congenital infection caused by cytomegalovirus (CMV), toxoplasmosis, rubella, or syphilis is associated with SNHL.
Hereditary (genetic) bilateral SNHL occurs in approximately 1 in 2000 births. In developed countries, it accounts for 50 to 60 percent of cases of SNHL in children. Approximately one-third are syndromic and two-thirds nonsyndromic. Approximately 80 percent of cases of HHL are inherited in an autosomal recessive pattern.
Mutations in the GJB2 gene that encodes the protein connexin 26 (CX26) on chromosome 13 cause nearly one-half of all bilateral severe to profound congenital hearing loss in nonsyndromic children.
The most common cause of mild to moderate nonsyndromic recessive hearing loss is mutation of the STRC gene. This gene encodes the protein stereocilin.
The autoantibody most specific for systemic lupus erythematosus is: A. antihistone. B. antinuclear. C. anti-ribonucleoprotein. D. anti-Ro (SS-A). E. anti-Sm.
E - anti-Sm
Lupus: RASH OR PAIN Rash (malar A or discoid) Arthritis (nonerosive) Serositis Hematologic disorders (eg, cytopenias) Oral/nasopharyngeal ulcers Renal disease Photosensitivity Antinuclear antibodies Immunologic disorder (anti-dsDNA, anti-Sm, antiphospholipid) Neurologic disorders (eg, seizures, psychosis)
Most sensitive = ANA (basically 100%)
Anti-dsDNA antibodies: Specific, poor prognosis (renal disease), useful for monitoring disease activity
Anti-Smith antibodies: Specific, not prognostic (directed against snRNPs)
Antihistone antibodies: Sensitive for drug-induced lupus (eg, hydralazine, procainamide)
In congestive cardiac failure, which one of the following factors is most important in the pathogenesis of peripheral
oedema?
A. Hypoalbuminaemia.
B. Impaired lymphatic drainage via thoracic duct.
C. Increased antidiuretic hormone production.
D. Increased capillary permeability.
E. Sodium retention
E - sodium retention
The predominant cause of nodule formation in a positive tuberculin skin test reaction is infiltration with: A. CD4+ T lymphocytes. B. CD8+ T lymphocytes. C. eosinophils. D. macrophages. E. neutrophils.
D - macrophages
An antenatal ultrasound reveals a right-sided thoracic lesion which appears to have a systemic blood supply. A computerised tomography (CT) scan performed after birth reveals an abnormal right lower lobe with air-filled cysts of varying size. Thickening and collapse are present within the same areas. Except for a cough the child has been asymptomatic but breath sounds are reduced in this area.
Which of the following is the most likely diagnosis? A. Bronchogenic cysts. B. Congenital lobar emphysema. C. Cystadenomatoid malformation. D. Polyalveolar lobe. E. Pulmonary sequestration.
E - pulmonary sequestration. I think the key is the systemic blood supply, bronchogenic cyst is the only other answer with systemic blood supply (?can be either), and as below ?not demonstrable at birth.
Uptodate:
Bronchopulmonary sequestration (BPS), sometimes referred to simply as pulmonary sequestration, is a rare congenital abnormality of the lower airway. It consists of a nonfunctioning mass of lung tissue that lacks normal communication with the tracheobronchial tree and that receives its arterial blood supply from the systemic circulation. By definition, their arterial blood supply is from the systemic circulation.
In contrast with BPS, CPAMs are connected to the tracheobronchial tree and are supplied from the pulmonary circulation.
Bronchogenic cysts arise from anomalous budding of the foregut during development and represent part of the spectrum of bronchopulmonary foregut malformations. They can occur at any point throughout the tracheobronchial tree. Affected patients typically present during the second decade of life with recurrent coughing, wheezing (which may simulate asthma), and pneumonia.
Congenital lobar emphysema (CLE), also known as congenital alveolar overdistension, is a developmental anomaly of the lower respiratory tract that is characterized by hyperinflation of one or more of the pulmonary lobes. The most frequently identified cause of CLE is obstruction of the developing airway, which occurs in 25 percent of cases. The left upper lobe is affected most often (40 to 50 percent of cases).
Polyalveolar lobe gives rise to congenital lobar emphysema.
The following results are obtained from a two-day-old male infant. IgG 6.53 g/L [5.34-16.94] IgA <0.10 g/L [0-0.07] IgM <0.09 g/L [0-0.18] IgE <5 kU/L [<25] Haemoglobin 189 g/L [145-225] Red cell count 5.06 x 1012/L [4.00-6.60] Platelet count 326 x 109 /L [150-400] White cell count 11.5 x 109 /L [5.0-21.0] Differential: band forms 0.68 x 109 /L (6%) neutrophils 9.01 x 109 /L (78%) lymphocytes 0.10 x 109/L (1%) monocytes 1.37 x 109/L (12%) eosinophils 0.34 x 109/L (3%)
These findings are most consistent with which one of the following? A. IgA deficiency. B. Kostmann syndrome. C. Normal results. D. Severe combined immune deficiency. E. X-linked agammaglobulinaemia.
D - SCID. Key point here is you should have detectable lymphocytes at birth, not 0.1/1%/so low. Normal for IgM/IgA to be low/undetectable.
Osmosis:
Severe combined immunodeficiency is the most severe form of primary immunodeficiencies, and may results from an X-linked recessive defect in IL-2R gamma chain or an autosomal recessive defect in adenosine deaminase deficiency. This disorder results in B-cell and T-cell disorders, resulting in the failure to thrive, chronic diarrhea, thrush, and recurrent infections.
IgG normal d/t maternal third trimester placental transfer.
T cell count low/non existent. Expect low T cells, potentially normal B cells, but hypogammaglobulinaemia due to lack of T helper cells.
T cell receptor excision circles (TREC) low (indicates T cell maturation -> low).
IgA deficiency - most common primary immunodeficiency, isolated deficiency in IgA, mostly asymptomatic. MUST BE >4 YEARS OLD.
X-linked agammaglobulinaemia - deficiency in all immunoglobulins due to failure of maturation of B cells (Bruton’s tyrosine kinase mutation). T CELL MEDIATED IMMUNITY REMAINS IN TACT. Newborns typically asymptomatic.
Kostmann’s syndrome is a disease of the bone marrow where children are born without a type of white blood cell - neutrophil ( also called a granulocyte ) which are normally used to fight infection.
A 30-month-old boy presented five weeks ago with sudden onset of limp and fever, having been well previously.
Both knees were red and swollen, but rapidly improved without treatment and in 24 hours were back to normal.
His fevers persisted.
He developed an intermittent maculopapular rash over his trunk and thighs. He was often miserable and disliked being handled, particularly being touched on his neck.
On examination he is miserable, febrile (39°
C), has a few small lymph nodes in the neck and groin and his spleen can be tipped. Examination is otherwise completely normal.
Investigations: haemoglobin 103 g/L [110-140] white cell count 17.9 x 109/L [4-11] platelet count 641 x 109/L [150-400] ESR 38 mm/h Chest X-ray moderate cardiomegaly Cardiac ultrasound moderate cardiac effusion without tamponade.
Which one of the following is the one most likely diagnosis? A. Coxsackie virus infection. B. Kawasaki disease. C. Rheumatic fever. D. Systemic juvenile chronic arthritis. E. Systemic lupus erythematosus.
D - systemic juvenile chronic arthritis. ?Different diagnostic criteria as must be present for 6 weeks for juvenile idiopathic arthritis. ?Not sure any of the other answers would have splenomegaly…
Juvenile idiopathic arthritis, also known as juvenile rheumatoid arthritis, is the most common form of arthritis in children and adolescents. It is an autoimmune, non-infective, inflammatory joint disease of more than 6 weeks duration in children less than 16 years of age.
Systemic arthritis — Systemic arthritis (systemic JIA) defines the category of patients that were previously labeled Still’s disease.
Systemic arthritis is characterized by arthritis in one or more joints and fever of at least two weeks duration that is quotidian for at least three days. In addition, children must have one or more of the following findings: evanescent, erythematous rash; hepatomegaly or splenomegaly; lymph node enlargement; or serositis.
Coxsackie virus - ?shouldn’t cause such prolonged semiology.
Kawasaki - right age (6mo-5yrs). Fever >5 days, 4/5 of bilat conjunctivitis, oral changes (strawberry tongue), rash, peripheral changes (hyperemia, painful oedema), lymphadenopathy classically (>1.5cm).
Rheumatic fever: age 5-14, no clear URTI history, occurs 10d-6 weeks after URTI. Acute rheumatic fever manifests as a nonsuppurative inflammatory process affecting cardiac tissue (Figure 3), joints, subcutaneous tissue and the central nervous system. Major manifestations:
Carditis (including subclinical evidence of rheumatic valve disease on echocardiogram)
Polyarthritis or aseptic mono-arthritis or polyarthralgia
Sydenham chorea
Erythema marginatum
Subcutaneous nodules
SLE: Too young (rare <5-6, can exist but monogenic and less common, usually adolescent). Also doesn’t sound right / doesn’t meet criteria.
A six-week-old infant weighing 3300 g is referred for evaluation of tachypnoea and failure to thrive. He was born at term weighing 3000 g. He is tachypnoeic, mildly cyanosed and has tachycardia with normal volume pulses.
Ejection and mid diastolic murmurs are heard. His transcutaneous oxygen saturation is 87% and two-dimensional Doppler echocardiography indicates double inlet left ventricle (single ventricle) with unrestricted pulmonary and aortic outflows.
Which one of the following would be the most appropriate surgical management?
A. Bidirectional Glenn shunt (superior vena cava to right pulmonary artery anastomosis).
B. Modified Blalock shunt (systemic to pulmonary artery anastomosis).
C. Modified Fontan procedure (total cavopulmonary connection).
D. Pulmonary artery banding.
E. Ventricular septation.
D - pulmonary artery banding. Key is unrestricted pulmonary outflow/evidence of CCF/high PBF.
Park’s:
The purpose of the first-stage operation is to make patients acceptable candidates for bidirectional Glenn or hemi-Fontan operation. When there is no PS, pulmonary overcirculation can lead to pulmonary hypertension, which jeopardises future Fontan operation.
In patients with no PS and large PBF with resulting CHF and pulmonary oedema, PA banding may be done (carries high mortality rate ~25%). PA banding is only performed when the BVF is normal or unobstructed.
If PS or PA is present and oxygen saturation <85%, a BT shunt is necessary to improve cyanosis.
Bidirectional Glenn at 3-6 months. Fontan at 18-24 months.
A four-year-old boy presents with a four-week history of screaming out in his sleep at approximately 11 p.m.each
night. When his parents attend to him he is usually sitting up in bed, eyes wide, highly agitated and muttering to
himself in a frightened way. His hair is dishevelled, he is breathing quickly and his arms and legs are shaking. He
does not appear to recognise his parents and is not consoled by their presence. On several occasions the boy has got up from his bed and run screaming from the house into the neighbours’ front yard in a highly agitated
state. On two occasions he has stood and urinated in the corner of his bedroom. His agitation usually subsides
after 10-15 minutes, following which the boy seems to settle back into a peaceful sleep.
The parents consult you, as their family paediatrician, about this behaviour.
The most appropriate initial management would be:
A. carbamazepine.
B. imipramine.
C. psychiatric referral.
D. reassurance that this behaviour will settle without treatment.
E. scheduled waking of the child at 10 p.m.
E - scheduled waking of the child at 10pm
Uptodate:
Parasomnias are episodic behaviors that intrude onto sleep and often lead to significant worry for the parents or the patient. The events are most common in preschool-aged children and gradually decrease in frequency over the first decade of life.
- Sleep terrors – 40 percent (100% remission rate in one study)
●Sleepwalking – 15 percent
●Sleep enuresis – 25 percent
●Bruxism (teeth grinding) – 46 percent
●Rhythmic movement disorder (eg, head banging) – 9 percent
Sleep terrors (often called night terrors) typically occur between 4 and 12 years of age. The events occur during the first one-third of nocturnal sleep. The child awakens abruptly from sleep with a loud scream, is agitated, and has a flushed face, sweating, and tachycardia. The child may jump out of bed as if running away from an unseen threat and is usually unresponsive to parental efforts at calming. The child usually does not remember the episode later. A simultaneously obtained EEG may show high-amplitude rhythmic delta or theta activity. There is a strong genetic predisposition.
Infrequently occurring (one to two times per month) confusional arousals, sleep terrors, and sleepwalking do not need to be treated.
Anticipatory awakening (scheduled awakening) is a behavioral technique that can be utilized to prevent these parasomnias. Anecdotally reported to have good effect. Although the efficacy of scheduled awakenings has not been well tested in clinical studies, the technique may be worth trying when the family is reluctant to administer medications and is inclined towards nonpharmacologic management. It has the potential drawback of actually triggering a parasomnia by the awakening procedure.
If no specific underlying triggers are found and the parasomnias remain problematic, we suggest a low dose of a benzodiazepine.
You are asked to review a 15-year-old boy regarding his short stature. He was treated for medulloblastoma at age
six with cranio-spinal irradiation. His height was on the 50th percentile at diagnosis. At age 11, when puberty was first noticed, his height was 140 cm (25th percentile). He is now 156 cm tall and his arm span is 167 cm. His
father’s height is 172 cm and his mother’s height is 158 cm.
Preliminary investigations include:
bone age 15 years
free thyroxine (free T4) 9 pmol/L [8-18]
thyroid-stimulating hormone (TSH) 9 mU/L [<4]
insulin-like growth factor 1 (IGF-1) 15 pmol/L [20-60]
The major cause of his short stature is: A. attenuated pubertal growth spurt. B. attenuated spinal growth. C. familial short stature. D. growth hormone deficiency. E. hypothyroidism.
B - attenuated spinal growth
Medulloblastoma - a non-glial embryonal brain tumour, cerebella, only in posterior fossa. 20% of CNS paed tumours and most common malignant brain tumour.
Endocrine morbidities are the most prominent disorder among the spectrum of long-term conditions, with growth hormone deficiency the most common endocrinopathy noted, either from tumor location or after cranial irradiation and treatment effects on the hypothalamic/pituitary unit.
Disproportionate growth after craniospinal radiation therapy (CSRT) is well recognized with notable high risk for short adult height. Spinal irradiation most likely causes damage to the growth plates, as patients treated with spinal RT are consistently shorter than patients treated with cranial RT despite treatment with GH.
Attenuated pubertal growth spurt - bone age is correct and arm span also seems to be growing as expected for puberty growth spurt, specifically spinal growth that seems affected.
Familial short stature = the parents are short -> the child is short. Wouldn’t expect dropping centiles, and should attain mid-parental height. Essentially just a variant of normal, but ya short.
GH deficiency - Children with acquired GHD present with severe growth failure, delayed bone age, and increased weight:height ratios. Causes of acquired GHD include intracranial tumors involving the hypothalamic-pituitary region (eg, craniopharyngioma), cranial irradiation, and head trauma.
Hypothyroidism - T4 normal. No other features.
The most appropriate treatment for moderate hypertension following post-infectious glomerulonephritis is: A. amiloride. B. atenolol. C. captopril. D. chlorothiazide. E. frusemide.
E - furosemide
I think the implication is that this is during the period of fluid overload, i.e. during the acute GN or the recovery phase, therefore given the issue is sodium and water retention (and often hyperkalaemia from AKI) it’s appropriate to use furosemide.
Recent readings suggest loop diuretic + CCB as appropriate first line.
Amiloride: Potassium sparing diuretic - often hyperkalaemia is an issue in GN
Captopril: ACE inhibitor will worsen AKI
A 10-year-old girl is diagnosed with type 1 (insulin-dependent) diabetes mellitus. At time of diagnosis, which one of the following antibodies is most likely to be found in her serum? A. Adrenal antibody. B. Gliadin antibody. C. Glutamic acid decarboxylase antibody. D. Insulin auto-antibody. E. Thyroid microsomal antibody.
C - GAD antibody
Think the key here is “at time of diagnosis”. IAA first but disappear by diagnosis, GAD second.
Uptodate:
Insulin autoantibodies are often the first to appear in children followed from birth and progressing to diabetes and are the highest in young children developing diabetes.
IAA (insulin autoantibodies) were almost always the first to appear, with other antibodies (ICA, GAD, or IA-2) appearing later.
Nelsons:
IAAs are usually the first to appear in young children,
followed by glutamic acid decarboxylase 65 kDa, and later by tyrosine phosphatase insulinoma–associated 2 and zinc transporter 8 antibodies
Other:
Islet cell antigen (ICA): The first islet ‘autoantigen’ to be described. Now known to be a complex of autoantigens. Antibodies to ICA are present in 90% of type 1 diabetes patients at the time of diagnosis.
GAD: A constituent of the ICA antigen complex. Present in the 65 kDa form in the human islet. Also present in the central nervous system. GAD antibodies are present in 73% of type 1 diabetes patients at diagnosis.
Antibodies to insulin and pro-insulin, the biochemical precursor to insulin, are present at diagnosis in 23% and 34% of type 1 diabetes patients, respectively.
A woman develops chickenpox at 38 weeks gestation. Fourteen days later she delivers a full-term baby boy
vaginally. The baby weighs 3200 g and is covered in vesicular spots. He is clinically well and requires no resuscitation. Physical examination is otherwise normal.
What is the most appropriate management of this baby?
A. Give intravenous acyclovir.
B. Give oral acyclovir.
C. Give intramuscular zoster immune globulin.
D. Give intramuscular zoster immune globulin and intravenous acyclovir.
E. No treatment.
E - no treatment
ASID guideline.
Risk of subsequent fetal varicella syndrome (FVS)
following maternal chickenpox in pregnancy:
- < 12 weeks gestation: 0.55%
- 12-28 weeks gestation: 1.4%
- >28 weeks gestation: No cases of FVS reported
Perinatal Chickenpox Rx Infants:
- Maternal chickenpox > 7 days before delivery: No ZIG. No other interventions unless baby <28/40 or <1kg, in which case aciclovir. No isolation. Breast feed.
- Maternal chickenpox -7 to +2 days from birth: ZIG immediately, w/i 24 hours (up to 72) from birth. Discharge ASAP. No isolation. Breast feed.
- Maternal chickenpox +2 to +28 days after delivery: ZIG required, can be up to 10 days after maternal rash. Discharge ASAP. No isolation. Breast feed.
IF INFANT DEVELOPS CHICKENPOX:
- very preterm: IV aciclovir
- term: if severe disease / respiratory symptoms / ZIG given >24 hours after birth, aciclovir.
A four-year-old girl developed a sore throat with difficulty swallowing, dribbling, fever to 39°C, and puffiness of both upper eyelids. She was given oral penicillin V by her local doctor, but her symptoms persisted and she developed a generalised, fine, discrete, macular rash on her trunk and face.
Examination showed a fever of 39.5°C, exudative tonsillitis, the macular rash, bilateral upper eyelid oedema, and generalised cervical, axillary and inguinal lymphadenitis. Her spleen could be tipped. Her chest was clear.
Investigations show: haemoglobin 144 g/L [110-140] white cell count 14.4 x 109/L [4-11] platelet count 363 x 109/L [150-400] monospot negative urinalysis 1+ protein serum antistreptolysin-O titre (ASOT) 479 [<200]
Which of the following is the most likely diagnosis? A. Adenovirus infection. B. Epstein-Barr virus infection. C. Herpes simplex virus infection. D. Penicillin allergy. E. Scarlet fever
B - EBV
Rash not consistent with scarlet fever and ?should improve with pencillin. ASOT takes 4-6 weeks to peak. Lymphadenopathy, splenomegaly more c/w EBV.
Uptodate:
Scarlet fever (also known as “scarlatina”) is a diffuse erythematous eruption that generally occurs in association with pharyngitis. Development of the scarlet fever rash requires prior exposure to S. pyogenes and occurs as a result of delayed-type skin reactivity to pyrogenic exotoxin (erythrogenic toxin, usually types A, B, or C) produced by the organism.
The rash of scarlet fever is a diffuse erythema that blanches with pressure, with numerous small (1 to 2 mm) papular elevations, giving a “sandpaper” quality to the skin.
The presence of palatal petechiae, splenomegaly, and posterior cervical adenopathy are highly suggestive of IM, while the absence of cervical lymphadenopathy and fatigue make the diagnosis less likely.
Heterophile antibodies react to antigens from phylogenetically unrelated species. The Monospot is a latex agglutination assay using horse erythrocytes as the substrate. The false-negative rates are highest during the beginning of clinical symptoms (25 percent in the first week). Heterophile antibody tests are often negative in infants and children less than four years of age.
ASOT: Following streptococcal pharyngitis, the antibody response peaks at about four to five weeks.
Inhaled nitric oxide is usually delivered into the ventilation circuit close to the patient’s endotracheal tube.
This is in order to:
A. allow delivery of optimal concentrations of nitric oxide.
B. avoid exposure of nitric oxide to high humidification.
C. lessen the rate of generation of methaemoglobin.
D. minimise the reduction in FiO2 consequent on nitric oxide delivery.
E. minimise the risk of pulmonary toxicity due to other oxides of nitrogen.
E - minimise the risk of pulmonary toxicity due to other oxides of nitrogen
An intelligent eight-year-old boy has a long history of hyperactivity, impulsiveness and distractibility, together with motor tics (blinking, facial grimaces and snorting). He has been diagnosed as having Attention Deficit Hyperactivity Disorder (ADHD) and Tourette's disorder. He showed little response to clonidine (now ceased), but his hyperactivity improved with methylphenidate. He now presents with a six-month history of severe anxiety. He is unable to enter an empty room, cannot sleep alone and clings to his mother. At school he is functioning reasonably well in academic and social areas. At interview, he presents as tense and anxious and describes fears of monsters and dinosaurs entering his house and attacking him. He understands that these fears are baseless but cannot stop himself from worrying. There are no major precipitating events. You have commenced a behaviour modification program which is only partially successful and you decide to supplement methylphenidate with another medication.
Which one of the following medications is the most appropriate? A. Imipramine. B. Fluoxetine. C. Haloperidol. D. Moclobemide. E. Oxazepam
C - haloperidol (1999 question). Not sure about this one.
USMLE book
ADHD: Onset before age 12. Limited attention span and poor impulse control. Characterized by hyperactivity, impulsivity, and/or inattention in multiple settings (school, home, places of worship, etc). Normal intelligence, but commonly coexists with difficulties in school. Continues into adulthood in as many as 50% of individuals. Treatment: stimulants (eg, methylphenidate) +/– cognitive behavioral therapy (CBT); alternatives include atomoxetine, guanfacine, clonidine.
Tourette syndrome: Onset before age 18. Characterized by sudden, rapid, recurrent, nonrhythmic, stereotyped motor and vocal tics that persist for > 1 year. Coprolalia (involuntary obscene speech) found in only 10–20% of patients. Associated with OCD and ADHD. Treatment: psychoeducation, behavioral therapy. For intractable and distressing tics, high-potency antipsychotics (eg, fluphenazine, pimozide), tetrabenazine, guanfacine, and clonidine may be used.
Anxiety disorder: Inappropriate experience of fear/worry and its physical manifestations (anxiety) incongruent with
the magnitude of the perceived stressor. Symptoms interfere with daily functioning. Includes panic disorder, phobias, generalized anxiety disorder, and selective mutism. Treatment: CBT, SSRIs, SNRIs.
OCD: Recurring intrusive thoughts, feelings, or sensations (obsessions) that cause severe distress;
relieved in part by the performance of repetitive actions (compulsions). Ego-dystonic: behavior inconsistent with one’s own beliefs and attitudes (vs obsessive-compulsive personality disorder). Associated with Tourette syndrome. Treatment: CBT, SSRIs, and clomipramine are first line.
Antipsychotics uses: Schizophrenia (primarily positive symptoms), psychosis, bipolar disorder, delirium, Tourette
syndrome, Huntington disease, OCD.
A general practitioner requests your advice about a six-year-old boy who has sustained a laceration which has
been heavily soiled with manure after a fall in a horse stable. His immunisation status, confirmed by his child
health records, is as follows:
diphtheria-tetanus-pertussis vaccine (DTP) at two and four months
oral polio vaccine (OPV) at two and four months
measles-mumps-rubella vaccine (MMR) at 12 months
Which one of the following should the child now be given?
A. Childhood diphtheria and tetanus toxoids (CDT), tetanus immunoglobulin and OPV.
B. DTP.
C. DTP and OPV.
D. DTP, tetanus immunoglobulin and OPV.
E. Tetanus toxoid and tetanus immunoglobulin
D - DTP, tetanus immunoglobulin, OPV
Aus Imm Schedule:
DTP - 2/4/6/18 months + 4 years = 5 doses
OPV - 2/4/6 months + 4 years = 4 doses
This child is 6 years old with only 2x DTP and 2x OPV -> incomplete vaccination.
CDC guideline:
- if <3 DTP vax AND not a “clean, minor” wound -> DTP vax + TIG (if clean/minor just DTP no TIG)
- if >3 DTP vax -> nothing needed
Tetanus immunoglobulin ONLY for <3 doses vaccine and “all other wounds”, otherwise just vaccination if “all other wounds AND >5 years from last dose” or “>10 years since last dose with any wound”.
OPV presumably opportunistically due to incomplete vaccination.
A 10-day-old male infant, born at term with a birthweight of 2400 g is lethargic and feeding poorly. There has been no weight gain. An abdominal ultrasound demonstrates bilateral hydronephrosis. Urine output appears to be satisfactory. The micturating cysto-urethrogram is shown (see previous page). An indwelling catheter is left in situ.
The following biochemistry results are obtained the following morning:
Serum: sodium 126 mmol/L [132-145] potassium 6.9 mmol/L [3.5-6.0] chloride 88 mmol/L [100-110] HCO3 17.6 mmol/L [18.0-26.0] urea 15.5 mmol/L [1.4-4.6] creatinine 0.126 mmol/L [0.015-0.055] albumin 26 g/L [23-46] Urine: sodium 68 mmol/L potassium 24 mmol/L osmolality 195 mOsm/Kg
The primary objective in management over the next 24 hours would be which one of the following? A. Albumin replacement. B. Bicarbonate replacement. C. Potassium correction. D. Sodium replacement. E. Water restriction.
D - sodium replacement
Presuming this is a post obstructive diuresis / post AKI diuresis, clear losses of Na/K in urine, would expect K to normalise with diuresis and urinary losses, but patient already hyponatraemic and looking to lose lots of Na/H2O with diuresis.
A three-year-old boy presents with obesity, mild developmental delay and bilateral post axial polydactyly.
Corrective surgery to hands and feet was carried out in the first year. His photograph is shown (see previous
page). Ophthalmological assessment shows visual acuity is normal at 6/6 in both eyes. There is a pigmentary
retinopathy and the electroretinogram is abnormal.
The most likely diagnosis to explain these findings is: A. Bardet Biedl syndrome. B. Fragile X syndrome. C. hypothyroidism. D. Prader-Willi syndrome. E. Usher syndrome.
A - Bardet Biedl syndrome
Boast notes:
- Genetics + pathogenesis
a. 19 genes have been identified
b. AR
c. Inter-familial and intra-familial phenotypic variability - Clinical manifestations
a. Rode-cone dystrophy
i. Night blindness by 7-8 years
ii. Legal blindness by 15 years
b. Truncal obesity
i. Normal birthweight
ii. Rapid weight gain in first year
c. Postaxial polydactyly
d. Cognitive impairment + learning disability
e. Urogenital
i. Male hypogonadotropic hypogonadism
ii. Complex female genitourinary malformations
iii. Renal abnormalities - Prognosis
a. Poor prognosis
Fragile X: X-linked, ID+typical facies(prominent forehead, long narrow face, large ears)+macroorchidism
Prader-Willi:
o Floppy baby (hypotonia)
o Voracious appetite (hyperphagia)
o Obesity, short stature
o Moderate ID
o Hypogonadism
Usher: the most common genetic cause of combined vision and hearing impairment and deafblindness. People with Usher syndrome often have partial or total hearing loss and a gradual vision loss caused by retinitis pigmentosa.
Hypothyroidism doesn’t explain the polydactyly.
An eight-year-old boy presents with behavioural problems and a noted deterioration in his performance at school.
Examination reveals mild unsteadiness of gait and a slight brown discolouration of the gums. A computed
tomography (CT) scan of his head is shown below.
Which one of the following is most likely to establish a diagnosis?
A. Magnetic resonance imaging (MRI) scan of the brain.
B. Ophthalmological examination.
C. Plasma very-long chain fatty acid ratio.
D. Serum lead level.
E. White cell lysosomal enzymes.
C - plasma VLCFA ratio (elevated in nearly all peroxisomal disorders). This patient has X-linked adrenoleukodystrophy (hyperpigmentation of gums is the clue).
MRCPCH book:
Paediatric cerebral form of XLALD presents with severe neurological degeneration, usually between age 5 and 10, progressing to a vegetative state and death within a few years.
Features
- school failure, behavioural problems
- visual impairment
- quadriplegia
- seizures (late)
- adrenal insufficiency***
Adrenal involvement may precede or follow neurological symptoms by years. Some develop only neuro symptoms, others only have adrenal insufficiency. All boys developing adrenal failure should have VLCPFA checked to ensure this is not missed.
A four-year-old child presents to the emergency department with a two-day history of multiple bruises and a bleeding nose, two weeks after an upper respiratory tract infection. His past medical history is unremarkable. On examination, in addition to the features described in the history, there is a widespread petechial rash noted mainly over the trunk but there are no other abnormal features. Full blood count shows the following results:
haemoglobin 117 g/L [110-150] mean corpuscular volume 79 fL [75-90] red cell morphology normal platelet count <10 x 109/L [150-400] white cell count 9.8 x 109/L [5.0-14.5] differential: lymphocytes 5.8 x 109/L [1.5-10.0] neutrophils 3.8 x 109/L [1.0-8.0] monocytes 0.2 x 109/L [0.2-1.2]
Which one of the following treatments will result in the most rapid rise in the platelet count? A. Anti-D immunoglobulin. B. Danazol. C. Dexamethasone. D. Intravenous immunoglobulin. E. Prednisolone.
D - IVIG
Diagnosis ITP
- generally don’t need to treat as risk of bleeding low and given it’s a immunologic process any transfused platelets get mopped up immediately.
Out of date question (1999) but as per RCH guideline:
The decision to treat a child should be based on the clinical symptoms and not the platelet count, based on risk category.
Low (petechiae, bruises, crusted blood in nares): nil treatment, f/up 1 week with FBE
Moderate (epistaxis >5 mins, haematuria/chezia, painful purpura, menorrhagia): admit, haem to review film, 1st line pred or if needing RAPID RISE then IVIG (also 2nd line)
Severe/life threatening -> haem, IVIG+IVMP, platelet transfusion, tranexamic acid, +/- surg review
A 15-month-old boy presents with poor weight gain, lethargy, irritability and pallor. He has had five to six loose sloppy bowel motions a day for the last three weeks. Prior to this he was well. He is referred for a small bowel biopsy, which shows partial villous atrophy.
What is the most likely organism to have caused this clinical picture? A. Campylobacter jejuni. B. Enterotoxigenic E. coli. C. Rotavirus. D. Salmonella typhimurium. E. Vibrio cholerae.
C - rotavirus
Too long a duration for the others, not severe enough for cholerae. Age is right for rota as well (6mo-2yrs). Avg duration 8 days, but protracted illness can occur.
Causes of villous atrophy:
Autoimmune (coeliac disease, Crohns, AI enteropathy)
Infective (giardia, rotavirus/viral, HIV, TB)
Meds (Mycophenolate mofetil, azathioprine, methotrexate, and nonsteroidal antiinflammatory drugs)
Immunodeficiency (CVID)
Neoplastic
Ischaemic
Campylobacter: incubation 3 days, diarrhoea (bloody) fever abdominal pain vomiting, usually mild and self-limiting, diarrhoea last an average of 7 days and organisms may be shed for several weeks
ETEC: travellers diarrhoea, nearly always benign and self-limited, malaise, anorexia, and abdominal cramps followed by the sudden onset of watery diarrhoea, most resolved within 3-5 days
Salmonella typhimurium (non typhoidal): don’t cause enteric fever (typhi and paratyphi), incubation 8-72 hours, nonspecific gastro, generally self-limited 10-14 days.
Vibrio cholerae: acute secretory diarrhoeal illness caused by toxin production, severe cholera is characterized by profound fluid and electrolyte losses in the stool and the rapid development of hypovolemic shock, often within 24 hours from the initial onset of vomiting and diarrhoea. Incubation 1-2 days, the characteristic symptom of severe cholera (“cholera gravis”) is the passage of profuse “rice-water” stool, a watery stool with flecks of mucous. Abdominal discomfort, borborygmi, and vomiting are other common symptoms. Lasts 2-3 days.
A 17-year-old boy, repeating the second last year of secondary schooling, has been previously diagnosed by a paediatrician and a psychologist, as suffering from Attention Deficit Hyperactivity Disorder (ADHD) and a specific learning disorder. Central nervous system stimulants were helpful but discontinued because of poor sleep and volatile mood. The paediatrician is overseas and the boy now comes to you for help because he cannot study, is failing at school and he wants to go back on dexamphetamine.
He comes from a well functioning family in which there is no history of psychiatric illness or developmental
disorder. His parents report a progressive decline in his academic progress throughout secondary school and his
network of friends has been steadily reducing. He complains that his parents and teachers are too critical of him but admits that he often cannot be bothered studying and uses alcohol and marijuana but denies other drug use.
He says that he has not slept and has eaten little over the last few days. He worries that people are talking about him.
He looks unkempt, is agitated, restless and guarded. His affect is flat and you have trouble following his thought
processes. He denies that he is hearing voices.
What is the most likely cause of his latest presentation?
A. Attention Deficit Hyperactivity Disorder.
B. Bi-polar disorder (manic depressive psychosis).
C. Chronic marijuana abuse.
D. Depression.
E. Schizophrenia.
E - schizophrenia
USMLE book:
Chronic mental disorder with periods of
psychosis, disturbed behavior and thought,
and decline in functioning lasting > 6
months. Diagnosis requires at least 2 of the following,
and at least 1 of these should include 1–3
(first 4 are “positive symptoms”):
1. Delusions
2. Hallucinations—often auditory
3. Disorganized speech
4. Disorganized or catatonic behavior
5. Negative symptoms (affective flattening,
avolition, anhedonia, asociality, alogia)
ADHD: Onset before age 12. Limited attention span and poor impulse control. Characterized by hyperactivity, impulsivity, and/or inattention in multiple settings (school, home, places of worship, etc). Normal intelligence, but commonly coexists with difficulties in school. Continues into adulthood in as many as 50% of individuals. Treatment: stimulants (eg, methylphenidate) +/–
cognitive behavioral therapy (CBT); alternatives include atomoxetine, guanfacine, clonidine.
BPAD: Bipolar I defined by presence of at least 1 manic episode +/− a hypomanic or depressive episode.
Bipolar II defined by presence of a hypomanic and a depressive episode. Patient’s mood and functioning usually return to normal between episodes. Use of antidepressants can precipitate mania. High suicide risk. Treatment: mood stabilizers (eg, lithium, valproic acid,
carbamazepine), atypical antipsychotics.
Depression: May be self-limited disorder, with major
depressive episodes usually lasting 6–12 months. Episodes characterized by at least 5 of the following 9 symptoms for 2 or more weeks (symptoms must include patient reported depressed mood or anhedonia).
1. Depressed mood, 2. Sleep disturbance, 3. Loss of Interest (anhedonia), 4. Guilt or feelings of worthlessness, 5. Energy loss and fatigue, 6. Concentration problems, 7. Appetite/weight changes, 8. Psychomotor retardation or agitation, 9. Suicidal ideations
Treatment: CBT and SSRIs are first line.
SNRIs, mirtazapine, bupropion can also be
considered. Electroconvulsive therapy (ECT)
in select patients.
A 22-month-old girl presents with a hoarse voice, intermittent stridor, marked recession and fever. She has had four to five such episodes over the past six months but her voice has always been hoarse over this period.
The most likely diagnosis is: A. bronchogenic cyst. B. infantile larynx. C. laryngeal papillomatosis. D. lingual cyst. E. vascular ring.
C - laryngeal papillomatosis
Uptodate:
The prevalence of hoarseness in children ranges from 4 to 23 percent. Hoarseness can be caused by any process that affects the structure or function of the larynx. Aetiologic categories of hoarseness include infection, inflammation, trauma, obstruction or infiltration, and congenital anomalies.
Recurrent respiratory papillomatosis (RRP) is the most common benign laryngeal tumor in children. Benign tumors of the larynx are a rare cause of hoarseness in children. These tumors include papillomas, hemangiomas, cystic hygromas, and neurofibromas. RRP is typically diagnosed between the ages of two and three years. RRP is thought to be caused by acquisition of human papillomavirus (HPV) during passage through the birth canal of an infected mother.
Bronchogenic cyst: predominantly intrathoracic although can occur anywhere along tracheobronchial tree. Laryngeal cysts present in infancy. Affected patients (bronchogenic cysts) typically present during the second decade of life with recurrent coughing, wheezing (which may simulate asthma), and pneumonia
Infantile larynx = laryngomalacia
Lingual cyst: ?tongue not larynx
Vascular ring: should be a fixed stenosis
A nine-month-old boy, who has been fully immunised, presents with a pneumonia and has the following results: IgG 1.1 g/L [2.1-12.2] IgA <0.10 g/L [0.17-1.20] IgM 0.15 g/L [0.32-1.40] IgE <5 kU/L [0-35]
Lymphocyte markers: CD3 (T cells) 93% [53-71] CD4 (T helper) 68% [28-52] CD8 (T suppressor) 24% [13-31] CD19 (B cells) 0% [19-38] natural killer (NK) cells 4% [3-12]
His pneumonia is most likely to be due to which one of the following organisms? A. Escherichia coli. B. Haemophilus influenzae type b. C. Mycoplasma pneumoniae. D. Pneumocystis carinii. E. Staphylococcus aureus.
B - haemophilus influenzae type B
X-linked agammaglobulinaemia
- all Ig markedly reduced/absent
- B cells in peripheral blood absent/nearly absent (most characteristic finding)
Newborns asymptomatic and present after 6mo when placentally derived IgG no longer present.
AKA Bruton’s disease, defective Bruton’s tyrosine kinase, B cell precursors fail to mature.
There is only one characteristic physical finding of XLA and that is the absence or near absence of the B cell-rich tonsils and adenoids.
Recurrent bacterial respiratory tract infections are the most common manifestation of XLA. The infections are usually caused by encapsulated pyogenic bacteria, organisms for which opsonization by antibody is a primary host defense. ●Streptococcus pneumoniae ●Haemophilus influenzae type B ●Streptococcus pyogenes ●Pseudomonas species
A four-year-old boy presents with developmental delay and dysmorphic features. He is known to have a small
ventricular septal defect. The mother is 10 weeks pregnant. The parents are anxious to know if further children could be affected by the same condition. His speech is poorly articulated and difficult to understand. He has prominent ears, a high arched palate and a flat mid-face. The mother is described as a slow learner and has a repaired cleft palate. A Fluorescent in-situ Hybridisation (FISH) study performed on the child using a probe from the Di George critical region at locus 22q11 shows a deletion.
To assist in counselling these parents, the next most appropriate step is to:
A. arrange a fetal echocardiogram to be done at 24 weeks gestation.
B. arrange a FISH study on the boy using a probe specific for the Elastin gene at 7q11.23.
C. arrange a urinary metabolic screen on the child.
D. arrange for Fragile X cytogenetic studies on the child and his mother.
E. arrange for the parents to have a FISH study of locus 22q11.
E - arrange for the parents to have a FISH study of locus 22q11
Velocardiofacial syndrome
Need to know whether de novo mutation or whether inherited to determine risk.
Uptodate
Velocardiofacial syndrome (VCFS), also known as Shprintzen syndrome, is an autosomal-dominant disorder caused by a deletion in chromosome 22q11. This deletion has also been identified in the majority of patients with DiGeorge sequence and conotruncal anomaly face syndrome, suggesting that these are phenotypic variants of the same disorder.
The deletion occurs “de novo” in the majority of cases, but approximately 15 percent are inherited from an affected parent.
Affected patients have retrognathia, a long face with a prominent nose. Velopharyngeal incompetence (weak pharyngeal musculature) producing speech abnormalities such as hypernasal speech is very common. Submucous or overt clefts of the secondary palate can occur. Chronic otitis media is present in 75 percent of cases. Transient neonatal hypocalcemia occurs in 20 percent. Congenital heart defects occur in 85 percent of cases. The most common are ventricular septal defect (62 percent), right-sided aortic arch (52 percent), and tetralogy of Fallot (21 percent). Learning disabilities are frequent, and mild intellectual impairment occurs in approximately 40 percent of patients.
A four-year-old boy presents with a one-month history of easy bruising. Two weeks ago he was found to be
thrombocytopenic (platelet count 20 x 109
/L) and was treated with prednisolone. He has six café au lait spots, and has a repaired hypospadias. He has an asymptomatic ventricular septal defect.
His blood count is repeated: haemoglobin 108 g/L [100-130] mean corpuscular volume 97 fL [75-85] white cell count 7.5 x 109/L [5-15] (normal differential) platelet count 27 x 109/L [150-400]
Which one of the following is the most likely diagnosis?
A. Acute myeloid leukaemia.
B. Bernard-Soulier disease.
C. Chronic idiopathic thrombocytopenic purpura.
D. Fanconi anaemia.
E. Wiskott-Aldrich syndrome.
D - Fanconi anaemia
None of the other answers account for congenital heart disease and hypospadius.
Uptodate:
Fanconi anemia (FA) is an inherited bone marrow failure syndrome characterized by pancytopenia, predisposition to malignancy, and physical abnormalities including short stature, microcephaly, developmental delay, café-au-lait skin lesions, and malformations belonging to the VACTERL-H association.
Others
AML: WCC should be raised. Neoplastic monoclonal proliferation of myelogenous stem cells (myeloblasts) in bone marrow (>20%). Auer rods.
Bernard-Soulier: Defect in platelet plug formation. Large platelets. Reduced GpIb -> defect in platelet-to-vWF adhesion. Platelet count normal/low, increased bleeding time.
Chronic ITP: This ongoing form accounts for most ITP seen in adults and is far less common in children. Chronic ITP has similar symptoms to acute ITP, except that it lingers for longer than six months.
Wiskott-Aldrich: AKA eczema-thrombocytopenia-immunodeficiency syndrome. Most commonly X-linked, males, over 2 years old. Mutation in Wiskott-Aldrich syndrome protein (WASP) produced by all haematopoetic cells.
1999B.
A 14-year-old girl from interstate is seen in the Emergency Department with a fever. She has right thoracotomy and median sternotomy scars and her mother reports that she had heart operations at three and seven years of age. The chest X-ray obtained is shown below.
Her cardiac condition was most likely to have been:
A. persistent truncus arteriosus.
B. pulmonary atresia with intact ventricular septum.
C. tetralogy of Fallot.
D. transposition, ventricular septal defect and pulmonary stenosis.
E. tricuspid atresia.
D - transposition, VSD, PS
Unclear whether the age of operations, scars, or CXR hold the answer here.
Hypothalamic damage as a consequence of local tumours is not typically associated with which one of the following? A. Absent thirst. B. Excessive weight gain. C. Memory disturbance. D. Polydipsia. E. Tremor.
E - tremor
A 10 month old, exclusively breast fed boy is failing to thrive and has regressed in social and motor skills from age five months. Examination shows apathy, pallor, hypotonia with hyperreflexia, inability to sit unaided and unusual movements.
The following investigations are performed:
haemoglobin 70 g/L [100-130]
blood film macrocytosis
serum B12 50 ng/L [200-760]
serum folate 8 µ/L [3.1-7.1]
urine metabolic screen increased methylmalonic acid and homocystine
The most appropriate next step would be: A. bone marrow biopsy. B. enzyme assay in cultured fibroblasts. C. maternal serum B12 level. D. Schilling test. E. small bowel biopsy.
C - maternal serum B12 level
Vitamin B12 deficiency is uncommon in children but can occur in exclusively breastfed infants of mothers eating a strict vegetarian (vegan) diet, or with vitamin B12 malabsorption due to gastric bypass, short bowel syndrome, or pernicious anemia.
Increased methylmalonic acid levels are a sensitive indicator of mild vitamin B12 deficiency and elevated homocysteine levels denote vitamin B12 or folate deficiency.
- didn’t know this previously, basically makes the other options irrelevant.
Uptodate
Deficiencies of vitamin B12 and/or folate can cause megaloblastic anemia (macrocytic anemia with other features due to impaired cell division). Vitamin B12 deficiency can also cause neuropsychiatric findings. Human cells cannot synthesize vitamin B12 or folates. In most cases, a typical balanced diet will contain adequate amounts of both vitamins, with the notable exception that vegan diets do not contain adequate vitamin B12. Vitamin B12 (also called cobalamin) is present in many animal products, including meats, dairy products, and eggs. Many breakfast cereals are fortified with vitamin B12. In contrast, vitamin B12 is not present in foods derived from plants, with the exception of those that contain animal products or added vitamin B12. Vegans, strict vegetarians, and some pregnant or lactating women who limit animal protein can become vitamin B12-deficient.
Others:
- BM biopsy: too invasive for next line management and as above urine is suggestive of B12 deficiency
- enzyme assay in cultured fibroblasts: investigating for inborn errors of metabolism (specifically disorders of CHO, FA, mitochondria)
- Schilling test: The Schilling test is primarily of historical interest; this test is no longer routinely available. Prior to the availability of other testing for pernicious anemia, the Schilling test was used to determine if an individual with vitamin B12 deficiency was able to absorb vitamin B12 via the oral route.
- small bowel biopsy: coeliac disease, not relevant given exclusively breast fed, also IDA so microcytic
A 10-year-old girl is referred for evaluation of an intermittent fast heart beat occurring when watching television and after retiring. Her physical examination, chest X-ray, 12 lead ECG and echocardiograph are normal. The two illustrated tracings (see following page) are recorded from a simultaneous three channel Holter monitor system.
You would recommend treatment with which one of the following? A. Amiodarone. B. Flecainide. C. Propranolol. D. Sotalol. E. No drugs.
E - no durgs
?Looks like VT in one lead but not others -> inaccurate recording for that period? Hence the documentation with watching TV and going to bed, ?likely artifect.
OR it’s showing nonsustained VT with an otherwise normal 12 lead, exam, and TTE, which wouldn’t necessarily require treatment anyway if asymptomatic.
A 16-year-old male student anxiously presents to his doctor after his girlfriend noticed he was becoming yellow. He felt well and there were no abnormalities on physical examination. He denied drug or excessive alcohol use. He was a keen football player and had recently commenced a rigorous diet to prepare for the coming football season.
Blood investigations revealed: bilirubin -total 48 µmol/L [<10] -conjugated 7 µmol/L alanine aminotransferase (ALT) 25 U/L [10-50] gamma glutamyltransferase (GGT) 13 U/L [<45] alkaline phosphate 92 U/L [15-125] albumin 42 g/L [35-52] haemoglobin 128 g/L [120-160] white cell count 7.2 x 109/L [4.5-13] platelet count 155 x 109/L [150-400] film normal reticulocyte count 1% [<2%]
What investigation is appropriate? A. Epstein-Barr virus serology. B. Glucose-6-phosphate dehydrogenase screen. C. Hepatitis B serology. D. Liver ultrasound. E. No investigation.
E - no investigation
Isolated mild unconjugated hyperbilirubinaemia with normal LFTs otherwise, well patient, no evidence of haemolysis (normal film and retics), plus the evidence of stress/diet = Gilbert’s and requires no further work up.
Uptodate:
In a patient with a history consistent with Gilbert syndrome (eg, the development of jaundice during times of stress or fasting), normal serum aminotransferase and alkaline phosphatase levels and mild unconjugated hyperbilirubinemia (<4 mg/dL), additional testing is not required. Genetic testing can confirm the diagnosis in settings where there is diagnostic confusion.
The most common inherited disorder of bilirubin glucuronidation is Gilbert syndrome (also known as Meulengracht disease). Gilbert syndrome is a benign condition that has also been called “constitutional hepatic dysfunction” and “familial nonhemolytic jaundice”. Although many patients present as isolated cases, the condition can also run in families. It is characterized by recurrent episodes of jaundice and may be triggered by, among other things, dehydration, fasting, intercurrent disease, menstruation, and overexertion. Other than jaundice, patients are typically asymptomatic. The hyperbilirubinemia in patients with Gilbert syndrome is unconjugated. The diagnosis is made by excluding other causes of unconjugated hyperbilirubinemia, though genetic testing is available. No treatment is necessary, though it may be a risk factor for toxicity from some medications.
The prevalence of Gilbert syndrome has been reported to be between 4 and 16 percent.
Gilbert syndrome is the result of a defect in the promotor of the gene that encodes the enzyme uridine diphosphoglucuronate-glucuronosyltransferase 1A1 (UGT1A1), which is responsible for the conjugation of bilirubin with glucuronic acid.
A non-consanguineous couple has a son who is born with craniosynostosis, cleft palate and total (glove)
syndactyly involving all limbs. No one else in the family is similarly affected. His photograph is shown below. The
mother is 35 years old and the father is 46 years old.
The risk that their next child will be affected is closest to: A. 100%. B. 75%. C. 50%. D. 25%. E. 1%.
E - 1%
Referring to Apert syndrome - sporadic mutations. Boast notes:
- Apert syndrome
a. Sporadic (some AD)
b. FGFR2 mutation
c. Clinical manifestations
i. Skeletal - Irregular craniosynostosis (brachycephaly, plagiocephaly - coronal)
- Premature fusion of coronal, sagittal, lambdoid sutures
- Midface hypoplasia + OSA
- Exophthalmos but less severe than Crouzon
- Symmetrical syndactyly (fusion of 2nd/3rd/4th digits – single nail 2nd-4th digits)
ii. Neuro - Hydrocephalus
- Intellectual disability
Uptodate:
Apert syndrome is an autosomal dominant disorder that occurs in 6 to 15.5 out of 1 million livebirths. Most cases are sporadic. Mutations in the gene encoding fibroblast growth factor receptor 2 (FGFR2), located on chromosome 10, account for almost all known cases.
Syndactyly is a characteristic feature of Apert syndrome that permits distinction from other similar syndromes. Patients with Apert syndrome typically have a complex syndactyly known as mitten hand.
1999B
A one-year-old boy presents with vomiting, fever and lethargy. His abdomen is soft, kidneys are not palpable and his genitalia are normal. A suprapubic urine specimen contains 100 x 106 white blood cells/L and grows >108 E. coli/L in pure growth. He responds rapidly to intravenous ampicillin and gentamicin.
What investigations, if any, are indicated?
A. DMSA scan alone.
B. Micturating cystourethrogram and renal ultrasound.
C. No investigations unless he has a second infection.
D. Renal ultrasound scan alone.
E. Renal ultrasound and DTPA scan.
Outdated question. Answer was B (MCUG and USS) but as per RCH guidelines wouldn’t recommend this now. Answer should be C. Also in my experience on renal at RCH MCUG would only be done if USS showed dilation +/- for other concerns like atypical/recurrent UTIs.
RCH:
Other investigations
- Check renal function and consider renal ultrasound if the child is seriously unwell, or not responding to appropriate therapy after 48hrs
- Consider blood culture and lumbar puncture for unwell infants less than 4 weeks old, or if sepsis or meningitis is suspected at any age
Follow up
- Seriously unwell children, those with renal impairment, and boys <3 months of age should have a renal ultrasound prior to discharge
- Other children do not require an ultrasound for a first UTI; a non-urgent renal ultrasound should be arranged for children who have recurrent UTIs
- Routine antibiotic prophylaxis after simple UTI is not required
- Specialist follow-up should be arranged for children with recurrent UTI or known renal anomalies
The following results are obtained from a 14-year-old boy, prior to elective hernia surgery, who gives a history of excessive bleeding after dental extraction.
Bleeding time 6 minutes [2-9]
Prothrombin time 12 seconds [12-14]
Prothrombin time-international normalised ratio (PT-INR) 1.0 [0.9-1.2]
Activated partial thromboplastin time (APTT) 56 seconds [26-35]
Fibrinogen 2.4 g/L [1.8-4.0]
Mixing the patient’s plasma with an equal volume of normal plasma normalises the APTT.
Which one of the following is the most likely diagnosis? A. Antiphospholipid antibody syndrome. B. Factor VII deficiency. C. Haemophilia. D. Recent aspirin ingestion. E. Von Willebrand disease.
C - haemophilia. Being male is probably the key here. If female then VWD.
USMLE book:
Haemophilia:
Intrinsic pathway coagulation defect.
A: deficiency of factor VIII inc PTT; X-linked recessive.
B: deficiency of factor IX inc PTT; X-linked recessive.
C: deficiency of factor XI inc PTT; autosomal recessive.
Macrohemorrhage in hemophilia—hemarthroses (bleeding into joints, such as knee A ), easy bruising, bleeding after trauma or surgery (eg, dental
procedures). Treatment: desmopressin + factor VIII concentrate (A); factor IX concentrate (B); factor XI concentrate (C).
VWD:
Intrinsic pathway coagulation defect: reduced vWF = increased PTT (vWF acts to carry/protect factor
VIII).
Defect in platelet plug formation: reduced vWF = defect in platelet-to-vWF adhesion.
Autosomal dominant. Mild but most common
inherited bleeding disorder. No platelet
aggregation with ristocetin cofactor assay.
Treatment: desmopressin, which releases
vWF stored in endothelium.
APLS: prothrombotic
Factor 7 = extrinsic pathway = PT/INR abnormal
Aspirin = no affect on coag studies
1999B
An 11-year-old girl is referred because of recurrent severe asthma and very high doses of inhaled steroids and repeated courses of oral steroids (eight months on oral steroids in the last 12 months). She complains of severe shortness of breath with minimal exercise. She has gained 7 kg in the past 12 months. She has missed four months of school in the past year because of respiratory illness. Her lung function tests are normal. During an exercise test which she terminated at six minutes of bicycling, she was noted to have ‘severe distress’ associated with stridor and inspiratory and expiratory wheezes. Her flow volume loops (tidal volumes) during exercise are shown above and exhibit a saw-tooth appearance.
The most likely diagnosis is: A. exercise induced asthma. B. obliterative bronchiolitis. C. psychogenic asthma. D. recurrent spasmodic croup. E. unstable asthma.
C - psychogenic asthma
?Outdated terminology - ?vocal cord dysfunction
Uptodate:
A so-called “saw-tooth” pattern has been described that consists of small, rapid oscillations in flow during both expiration and inspiration (figure 12). Detecting the saw-tooth pattern is subjective and artifacts in the resonance frequency of the recording equipment can have a similar appearance.
Conditions associated with a saw-tooth pattern on the flow-volume loop include neuromuscular diseases, Parkinson disease, laryngeal dyskinesia, pedunculated tumors of the upper airway, tracheobronchomalacia, upper airway burns, and obstructive sleep apnea (OSA).
Asthma: should show obstructive picture
Obliterative bronchiolitis: obstructive
Croup: ?Variable extrathoracic obstruction
A 26-year-old woman with a history of recurrent genital herpes presents at 38 weeks gestation in labour. She is
found to have a small, active herpetic lesion on her cervix uteri. A caesarean section is performed four hours after rupture of membranes. A healthy baby girl is born weighing 3600 g and examination of the baby is normal.
What would be the most appropriate management plan for this baby?
A. Commence oral acyclovir immediately.
B. Commence intravenous acyclovir immediately.
C. Take viral swabs from the baby’s conjunctiva and nasopharynx immediately.
D. Take viral swabs from the baby’s conjunctiva and nasopharynx 24-48 hours after delivery.
E. No action necessary, unless lesions develop.
D - viral swabs from conjunctiva and nasopharynx 24-48 hours after delivery
- outdated
- currently as below, guidelines aren’t specific about swabs, would be as per advice from ID
- D or E acceptable answers based on current guidelines
ASID:
For current HSV: If HSV detected in genital tract at delivery: risk of transmission is 1-3%; risk greater for HSV1 (15%) than HSV2 (<0.01%). Much higher risk (25-50%) if no past history and no seroconversion before 30-34 weeks.
Management algorithm - “Asymptomatic infant with
low risk of neonatal HSV disease i.e. mother with recurrent genital infection, or primary genital or systemic infection seroconverted well prior to delivery with
or without genital lesions at delivery”:
As transmission is rare in this setting, most experts follow infants for clinical disease and educate family about signs of infection. However, some experts recommend swabs (eye, throat, umbilicus, rectum +/- urine) and blood for HSV PCR at 24 hours of life or beyond to detect asymptomatic infection based on USA consensus guidelines. Note: swabs taken <24 hours are thought to indicate colonisation not infection.
If high risk (mother with primary genital or systemic
infection close to delivery or infant born through birth canal with active HSV disease to mother with no prior history of genital HSV):
Perform:
• LP (CSF analysis, viral culture, HSV PCR)
• blood count (for low platelets),
• LFT’s
• HSV PCR on blood (if available)
• surface swabs, eye, throat, umbilicus, rectum, urine (if not collected already), skin vesicle.
Commence IV aciclovir immediately from birth (suspected HSV disease), duration will depend on surface
culture & CSF results.
The following investigations are performed on a 12-week term infant. IgG 3.08 g/L [1.45-11.28] IgA <0.10 g/L [0-0.55] IgM 0.74 g/L [0.21-0.88] IgE 5 KU/L [0-45] Whole blood PHA response 49% [70-150] CD3 42% [53-75] CD4 8% [23-60] CD8 34% [14-25] CD19 23% [12-39] Natural killer cells 15% [3-9] White cell count 17.7 x 109 /L [5.0-19.5] Red cell count 4.6 x 1012/L [2.7-4.9] Haemoglobin 116 g/L [90-140] Platelet count 238 x 109 /L [150-400] Mean corpuscular volume 77 fL [77-115] Mean corpuscular haemoglobin 26 pg [26-34] MANUAL DIFFERENTIAL Band forms 0.18 x 109 /L (1%) Neutrophils 11.86 x 109 /L (67%) [1.0-9.0] Lymphocytes 4.78 x 109/L (27%) [2.5-16.5] Monocytes 0.89 x 109/L (5%) [0.2-1.5]
These results are most consistent with: A. Di George syndrome. B. HIV infection. C. isolated IgA deficiency. D. severe combined immunodeficiency. E. Wiskott-Aldrich syndrome.
A - DiGeorge. Likely partial thymic aplasia, as complete aplasia should produce a SCID phenotype with T cell count low/non existent. Expect low T cells, potentially normal B cells, but hypogammaglobulinaemia due to lack of T helper cells.
Osmosis:
AKA 22q11.2 deletion. Deletion of TBX gene, impaired development of pharyngeal pouches 3 and 4, hypoplasia of thymus and parathyroid gland -> immunodeficiency (deficiency in mature T cells), hypocalcaemia, cardiac defects (truncus, TOF).
Facies: long face, small teeth, broad nose
Cleft palate
ID
T cell function proliferation assays: PHA (phytohemagglutinin) is a mitogen allows measurement of peripheral blood T cell proliferation in response to stimulus. Mitogens are powerful T cell stimulants.
SCID: Defect in T and B cell development, severe, causes vary but cytokine receptor defects (IL2-R) and adenosine deaminase deficiency most common (intracellular accumulation of lymphocyte toxic metabolites).
HIV: Specific for CD4
Isolated IgA: No as other lines suppressed
Wiskott-Aldrich: Not thrombocytopenic. Wiskott-Aldrich: Triad of microthrombocytopenia, eczema, recurrent pyogenic infections. X-recessive. Mutation in WASP gene (Wiskott-Aldrich syndrome protein).
A three-year-old girl develops otitis media and is treated with an antimicrobial agent. Eight days later she presents
with fever, an urticarial rash and painful swelling of her ankles and right knee. Examination shows a resolving otitis media and confirms there is some swelling and painful limitation of movement of the ankles and right knee. An urticarial rash is noted. She has a temperature of 38.5°C.
Which one of the following antimicrobial agents is most likely to have caused this clinical picture? A. Amoxycillin. B. Amoxycillin-clavulanic acid. C. Cefaclor. D. Cotrimoxazole. E. Erythromycin.
C - cefaclor
Serum sickness like reaction. Classic drug.
The most common signs and symptoms of serum sickness are dermatitis (rash), fever, malaise, and polyarthralgias or polyarthritis.
Uptodate:
The cardinal features of serum sickness are rash, fever, and polyarthralgias or polyarthritis, which begin one to two weeks after the first exposure to the responsible agent and resolve within a few weeks of discontinuation. Although patients may appear very ill and uncomfortable during the acute febrile stage, the disease is self-limited, and prognosis is excellent once the responsible drug is stopped.
Serum sickness is the prototypic example of the Gell and Coombs “type III” or immune complex-mediated hypersensitivity disease.
SS is generally the result of administration of a protein antigen from a nonhuman species, e.g. monoclonal antibodies.
Reactions to a variety of drugs (especially cefaclor and anti-seizure medications) can clinically resemble classic serum sickness but are believed to be caused by different mechanisms. These are called serum sickness-like reactions (SSLRs).
Serum sickness is more common in adults, while SSLRs are more common in children.
A 12-year-old girl is referred with a provisional diagnosis of absence epilepsy. On specific questioning she
expresses concern that she often shakes and drops objects, particularly in the mornings. There is no family
history of epilepsy. Neurological examination is normal. School progress is satisfactory. An EEG reveals 4-6 Hz
bilaterally symmetrical polyspike and wave patterns with normal background.
Which one of the following is the most likely diagnosis?
A. Benign occipital epilepsy.
B. Benign partial epilepsy of childhood with Rolandic spikes.
C. Childhood absence epilepsy.
D. Complex partial epilepsy.
E. Juvenile myoclonic epilepsy.
E - Juvenile myoclonic epilepsy
Uptodate:
Juvenile myoclonic epilepsy (JME) is frequently diagnosed in pediatric epilepsy clinics but is often not recognized by referring clinicians. Typically, the patient is a healthy young teenager with one or more of the following seizure types:
●Myoclonic jerks (most frequent in the morning, within the first hour after awakening)
●Absence seizures (“typical” petit mal seizures that often precede the other seizures and begin toward the end of the first decade)
●Generalized tonic-clonic seizures, which also have a tendency to occur upon awakening
Childhood absence epilepsy – Childhood absence epilepsy is an idiopathic generalized epilepsy with a peak age of onset of six to seven years, affecting girls more than boys, characterized by frequent (multiple per day) absence seizures in an otherwise normal child. GTCS often develop in adolescence. Myoclonic jerks do not occur in CAE.
Benign occipital epilepsy of childhood (Gastaut type) produces frequent seizures with prominent visual symptoms (hallucinations, blindness). The mean age of onset is between eight and nine years. EEG reveals occipital spikes that are activated by eye closure.
The following information was obtained at cardiac catheterisation in a six-month-old infant with a loud systolic murmur at the lower left sternal edge:
Essentially shows a VSD with pulmonary HTN
- sats normal until RV where jump from 69 to 88
- pulmonary artery pressures elevated, and RV pressures elevated
- left side normal
Which one of the following additional auscultatory murmurs is most likely to be heard?
A. A continuous murmur at the upper left sternal edge.
B. A mid diastolic murmur at the apex.
C. A mid diastolic murmur at the lower left sternal edge.
D. An early diastolic murmur at the upper left sternal edge.
E. An ejection systolic murmur at the upper left sternal edge
C - mid diastolic rumble
Park’s cardiology:
An apical diastolic rumble is present with a moderate-large shunt, caused by an increased flow through the mitral valve during diastole.
A five-year-old boy is seen with a history of recurrent wheezing episodes. His parents have managed these
episodes by intermittent use of nebulised bronchodilators and when away from home have used his mother’s “Ventolin” inhaler. The parents have agreed to a trial of preventative medication.
The most appropriate method of administration would be:
A. by aerosol alone with careful parental supervision.
B. by use of aerosol via a large volume spacer.
C. by use of aerosol via a small volume spacer with mask.
D. by use of dry powder inhalation.
E. by use of the nebuliser.
B - aerosol via large volume spacer
1999B - outdated
As per RCH guideline:
Spacers
- A spacer device should be used for children of all ages whenever they use a MDI
- Small volume spacers are suitable for children of all ages
- Small volume spacers should be fitted with a well-sealing face mask for younger children who cannot reliably seal their lips around the mouthpiece (generally <4-5 years old)
In an asymptomatic person with human immunodeficiency virus (HIV) infection, which one of the following is the best predictor of the future rate of decline of immune function?
A. CD4+ lymphocyte count. B. CD8+ lymphocyte count. C. p24 antigenaemia. D. Plasma HIV RNA concentration. E. Serum β2 microglobulin concentration.
D - plasma HIV RNA concentration
Uptodate:
Most untreated HIV-infected children show progression of HIV infection to acquired immune deficiency syndrome (AIDS) and die before age five years. In children, the most common OIs are serious bacterial infections, such as pneumonia and bacteremia, followed by herpes zoster, disseminated M. avium complex infections, and invasive candidal infections of the esophagus or tracheobronchial tree.
1996 paper:
Current clinical wisdom holds that the prognosis of HIV-infected persons – including their need for antiretroviral therapy and other medications – is best assessed by their CD4 lymphocyte counts. However, mounting evidence suggests that PCR measurement of plasma HIV RNA (“viral load”) is actually a far more powerful tool for this purpose.
A five-year-old girl with a two-year history of steroid responsive nephrotic syndrome has experienced four relapses over this period. Three months ago she relapsed and was treated with oral prednisolone in a dose of 1 mg/kg/day. Her oedema resolved but her proteinuria on albustix testing persisted at 2+. The prednisolone dose has been tapered to 0.5 mg/kg on alternate days. She now presents with generalised oedema and gross ascites with 4+ proteinuria. A renal biopsy was performed.
?MCD, who knows
Which one of the following is the most appropriate therapeutic response?
A. An angiotensin converting enzyme inhibitor.
B. Azathioprine.
C. Cyclophosphamide.
D. Cyclosporin.
E. Prednisolone 2 mg/kg/day.
E - pred 2mg/kg/day. Steroid responsive, but now frequent relapse/dependent.
I guess the point here is she has steroid responsive disease, and is presenting with a relapse. Therefore acute management is pred.
Frequent relapsing: defined as four or more relapses per year or two or more relapses in the initial six months after diagnosis. This girl now meets the criteria, as she has had 2 relapses in 3 months. In addition, there is a group of patients who are steroid dependent (defined as relapsing during taper or within two weeks of discontinuation of steroid therapy).
For longer term management nephrology team would probably consider a calcineurin inhibitor, and at RCH tacrolimus seems to be the second line agent of choice to get off pred.
A three-month-old infant is recognised as having an early systolic murmur, grade 3/6 in intensity at the lower left sternal edge and towards the apex. It starts abruptly with the first heart sound and disappears before the second. Examination is otherwise normal.
The most likely diagnosis is: A. atrial septal defect. B. tetralogy of Fallot. C. peripheral pulmonary artery stenosis. D. pulmonary valve stenosis. E. small muscular ventricular septal defect.
E - small muscular VSD
Park’s:
A grade 2-5/6 systolic murmur is heart at the LLSB. It may be holosystolic or early systolic. An apical diastolic rumble is present with a mod-large shunt due to increased flow through the mitral valve during diastole.
ASD: generally have a flow murmur across the pulmonary valve due to relative stenosis (from increased blood flow). Murmur is systolic, maximal at LUSB.
TOF: Murmur present from birth. Cyanotic. RV tap and systolic thrill commonly present. Long, loud, ejection systolic murmur at mid and upper LSB due to PS, plus holosystolic murmur of VSD.
PPAS: Midsystolic murmur in the pulmonary valve area, transmitted to axillae and back.
PVS: As for ASD
A 10-year-old girl with common variable immunodeficiency presents with a five-day history of cough, fever and shortness of breath.
Which one of the following organisms is most likely to be the cause of her respiratory symptoms? A. Cytomegalovirus. B. Mycobacterium avium complex. C. Pneumocystis carinii. D. Pseudomonas aeruginosa. E. Streptococcus pneumoniae.
E - Streptococcus pneumoniae
Osmosis:
CVID: hypogammaglobulinaemia due to defect in B cell differentiation, impaired differentiation into plasma cells.
Encapsulated bacteria: Haemophilus influenzae, pneumococci, streptococci
Which one of the following would be least appropriate for the assessment of a 36-month-old infant with
suspected neuro-developmental delay?
A. Bailey scale.
B. Griffith’s scale.
C. McCarthy scale.
D. Peabody picture vocabulary test.
E. Wechsler intelligence scale for children.
2000A Paper
E - WISC
Wechsler: 6-16years Involves: • Verbal comprehension • Visual spatial • Fluid reasoning • Working memory • Processing speed Good points • Generates an IQ score • Can be used to dx ADHD and learning disability • 45-65 minutes to administer Limitations • Does NOT include reading ability
Others Bailey: 1-3 years, gold standard and good for severe delay Griffiths: 0-2yrs McCarthy: 2-8years Peabody: >2.5years
The mean pulmonary artery (capillary) wedge pressure most closely approximates which one of the following pressures? A. Left atrial mean. B. Pulmonary artery diastolic. C. Pulmonary vein wedge mean. D. Right atrial mean. E. Right ventricular end-diastolic.
A - left atrial mean
Which area of the lung has the highest ventilation/perfusion (V/Q) ratio in the erect position? A. Anterior. B. Diaphragmatic. C. Middle. D. Posterior. E. Upper.
E - upper
Gravity dependent
Ventilation higher in elevated area
Perfusion greater in lower area
-> V/Q highest in upper portion whilst erect
In acute lymphoblastic leukaemia, which cytogenetic feature carries the worst prognosis? A. Hyperdiploidy. B. Normal chromosome number. C. Translocation (1;19). D. Translocation (9;22). E. Translocation (12;21).
D - translocation 9;22
Philadelphia chromosome
Poor prognosticators:
- age (<2 or >9)
- WCC >50
- boys
- chromosomal abnormalities/translocations
- hypodiploidy (near haploploidy carries the worst prognosis)
- CNS disease
Which one of the following autoantibodies is most specific for coeliac disease? A. IgA endomysial. B. IgA gliadin. C. IgA gluten. D. IgA reticulin. E. IgG gluten.
A - IgA endomysial
Question from 2000A - ?outdated
Current screening/serology includes IgA TTG, IgG DGP, +/- total IgA (to screen for IgA deficiency which creates false negatives for TTG IgA).
I known IgA deficiency can test TTG with IgG.
DGP replaced the old anti-gliadin test due to improved specificity.
Uptodate:
Approximately 2 percent of children with celiac disease will have previously unrecognized IgA deficiency. Although European guidelines recommend routine measurement of serum total IgA in all patients, we think a more cost-effective population approach is to measure total IgA in those children with negative results of tTG-IgA but a high clinical suspicion of celiac disease. For all other children, the yield is very low.
For most patients, the most valuable test is for IgA antibodies against tissue transglutaminase (tTG-IgA). This test is highly sensitive, specific, and more cost-effective than other antibody tests. The sensitivity and specificity of tTG-IgA antibodies for biopsy-proven celiac disease are generally above 96 percent (lower in children <2).
Testing for anti-endomysial antibodies (EMA) is as accurate as tTG-IgA, but this test is more expensive and somewhat dependent on operator interpretation. As a result, EMA is typically used as a second-line test to clarify the diagnosis in patients with equivocal results of tTG-IgA, including asymptomatic members of a high-risk group.
Deamidated gliadin peptide (DGP) also has good diagnostic accuracy and may be particularly useful for young children; this is a second-generation anti-gliadin antibody test.
Tests of anti-reticulin antibodies have reasonably high specificity, but lower sensitivity, and are no longer commonly used.
Standard (first-generation) IgA or IgG anti-gliadin antibodies are considerably less reliable and are not recommended.
A 12-month-old girl is referred to you for assessment and advice/management of her obesity.
Her birthweight was 3600 g (75th percentile) and her length was 51 cm (75th percentile). Now, her weight is
12.5 kg (1 kg >97th percentile) and her length is 78 cm (90th percentile). Neither parent is obese.
Developmentally she is normal and you can find no pathological reason for her obesity.
What is the approximate likelihood that she will be obese in her 20s? A. 15%. B. 30%. C. 45%. D. 60%. E. 75%.
A - 15%
A male infant has pulmonary atresia/ventricular septal defect. After undergoing a systemic to pulmonary
artery shunt his arterial saturation is 90%. Assuming a mixed venous saturation of 70% and a pulmonary
venous saturation of 95%, the pulmonary to systemic flow ratio (Qp:Qs) is:
A. incalculable without a known haemoglobin.
B. incalculable without a known oxygen consumption.
C. 0.2:1.
D. 1:1.25.
E. 4:1.
E - 4:1
Qp/Qs = (arterial-mixed venous sats)/(pulmonary venous sats - pulmonary artery sats)
?Because systemic->portal shunt i.e. bypassing right side of heart, only output into PA will be from the left-right shunt through the VSD -> will be same as arterial?
= (90-70)/(95-90)
= 20/5
= 4/1
Your region has a birth rate of about 50,000 babies per year. About 50 preschool children are diagnosed
each year with “crumblitis”, a rare gastrointestinal disorder, the cause of which is unknown. You hypothesise that this condition is related to maternal ingestion of orange rhubarb during pregnancy.
Which one of the following is the most appropriate research design to test your hypothesis?
A. Case-control study.
B. Cohort study.
C. Own control (crossover) trial.
D. Prevalence (cross-sectional) survey.
E. Randomised controlled trial.
A - case control study
Which one of the following vaccines is most likely to be damaged by freezing?
A. Haemophilus influenzae type b conjugate.
B. Hepatitis B.
C. Measles-mumps-rubella.
D. Meningococcal polysaccharide.
E. Oral polio.
B - hepatitis B
DPT, hepatitis B and tetanus toxoid vaccines can be damaged by freezing.
Which one of the following is the most common cause of familial thrombophilia (deep vein thrombosis and pulmonary emboli)? A. Activated protein C resistance. B. Antithrombin III deficiency. C. Protein C deficiency. D. Protein S deficiency. E. Prothrombin gene mutation.
A - activated protein C resistance.
The most common cause of APCR is factor 5 leiden.
Uptodate
The most common ITs identified in pediatric VTE include factor V Leiden, prothrombin G20210A mutation, antithrombin deficiency, and proteins S and C deficiencies. FVL accounts for 5-10%.
Factor V Leiden (FVL) results from a point mutation in the F5 gene, which encodes the factor V protein in the coagulation cascade. FVL renders factor V (both the activated and inactive forms) insensitive to the actions of activated protein C (aPC), a natural anticoagulant. As a result, individuals who carry the FVL variant are at increased risk of venous thromboembolism (VTE).
In most cases of pediatric VTE, there are underlying risk factors, the most common of which is the presence of an indwelling central venous catheter. Inherited thrombophilias (IT) also contribute to the risk of VTE; however, the prevalence of IT varies considerably depending upon the specific patient population. An underlying thrombophilia is far more likely in patients with unprovoked VTE compared with provoked (eg, catheter-related).
The incidence of pediatric VTE is highest among hospitalized neonates and infants, but there is a second “peak” in adolescence often in the setting of oral contraceptive use.
Curve A represents the log dose-response curve for drug X when given alone.
Curve B represents the log dose-response curve for drug X in the presence of a fixed concentration of another drug, Y.
(Curve B essentially half the height of curve A)
The most likely explanation for these findings is that:
A. X is a competitive antagonist of Y.
B. X is a non-competitive antagonist of Y.
C. Y is a competitive antagonist of X.
D. Y is a non-competitive antagonist of X.
E. Y is a partial agonist.
D - Y is a non-competitive antagonist of X
An eight-year-old boy presents with a presumed stroke. He has a profound right hemiparesis involving the
face, arm and leg.
Which one of the following clinical signs would suggest that the cerebral cortex is involved?
A. Aphasia.
B. Dense sensory loss to pinprick and touch.
C. Dystonic posture.
D. Equal involvement of the face, arm and leg.
E. Eye deviation.
A - aphasia
Google:
Cortical symptoms or signs include aphasia, agraphia, alexia, acalculia, neglect, extinction, apraxia, agnosia (including cortical sensory loss such as astereognosis), and hemianopia.
Strokes affecting the cerebral cortex (i.e. cortical strokes) classically present with deficits such as neglect, aphasia, and hemianopia. Subcortical strokes affect the small vessels deep in the brain, and typically present with purely motor hemiparesis affecting the face, arm, and leg.
In which one of the following is an increased ionised serum calcium most likely to occur? A. Alkalosis. B. Autoimmune polyglandular syndrome. C. Hypoalbuminaemia. D. Vitamin D deficiency. E. Williams syndrome.
E - Williams syndrome
Pastest book:
Calcium in the body is 40% protein bound (98% bound to albumin), 48% ionized, and 12% complexed to anions such as phosphate or citrate.
Williams
- heterozygous deletions of chromosomal subband 7q11.23 leading to an elastin gene defect in >90%. Hypercalcaemia rarely persists beyond 1 year.
Other
Alkalosis - degree of protein binding of plasma calcium is proportional to plasma pH. Increased pH leads to increased protein-bound calcium which reduces ionised calcium.
APS - Type 1 = HYPOparathyroidism, addisons, chronic mucocutaneous candidiasis, type 2 = HYPOparathyroidism, primary hypogonadism, T1DM, pernicious anaemia, addisons, vitiligo
Hypoalbuminaemia - will reduce total calcium concentration (see above) but doesn’t seem to affect ionised calcium.
Vit D deficiency - hypocalcaemia d/t reduced absorption